Brilliant problem clarification

Hello all, I have a little problem. The problem is deal with the current Brilliant problem about number theory.

Here's the link: https://brilliant.org/community-problem/mind-your-3s-and-2s/?group=69VNhEAzD53X

My question is Should m and n always different, or are they can be same? Thanks

#Brilliant.org

Note by Leonardo Chandra
7 years, 5 months ago

No vote yet
1 vote

  Easy Math Editor

This discussion board is a place to discuss our Daily Challenges and the math and science related to those challenges. Explanations are more than just a solution — they should explain the steps and thinking strategies that you used to obtain the solution. Comments should further the discussion of math and science.

When posting on Brilliant:

  • Use the emojis to react to an explanation, whether you're congratulating a job well done , or just really confused .
  • Ask specific questions about the challenge or the steps in somebody's explanation. Well-posed questions can add a lot to the discussion, but posting "I don't understand!" doesn't help anyone.
  • Try to contribute something new to the discussion, whether it is an extension, generalization or other idea related to the challenge.
  • Stay on topic — we're all here to learn more about math and science, not to hear about your favorite get-rich-quick scheme or current world events.

MarkdownAppears as
*italics* or _italics_ italics
**bold** or __bold__ bold

- bulleted
- list

  • bulleted
  • list

1. numbered
2. list

  1. numbered
  2. list
Note: you must add a full line of space before and after lists for them to show up correctly
paragraph 1

paragraph 2

paragraph 1

paragraph 2

[example link](https://brilliant.org)example link
> This is a quote
This is a quote
    # I indented these lines
    # 4 spaces, and now they show
    # up as a code block.

    print "hello world"
# I indented these lines
# 4 spaces, and now they show
# up as a code block.

print "hello world"
MathAppears as
Remember to wrap math in \( ... \) or \[ ... \] to ensure proper formatting.
2 \times 3 2×3 2 \times 3
2^{34} 234 2^{34}
a_{i-1} ai1 a_{i-1}
\frac{2}{3} 23 \frac{2}{3}
\sqrt{2} 2 \sqrt{2}
\sum_{i=1}^3 i=13 \sum_{i=1}^3
\sin \theta sinθ \sin \theta
\boxed{123} 123 \boxed{123}

Comments

The integers m and n can be the same. The answer the website thinks is correct is actually wrong, so if you got the right answer and the website said that you were incorrect, that is why. Hopefully, the staff will fix this soon.

Jon Haussmann - 7 years, 5 months ago

Log in to reply

Thanks for your reply. I've just got the answer, but it's wrong. Jon, I wonder how can you solve this problem: https://brilliant.org/mathematics-problem/let-our-powers-combine/?group=vAjLywc9ZnIz,

I have already used all the try: http://tinypic.com/r/2dhcvtx/5

But, cannot get the correct answer. Maybe you can try to send me your solution via my e-mail? My e-mail: leonardochandra@hotmail.com. Thanks

Leonardo Chandra - 7 years, 5 months ago

Log in to reply

I don't really have a solution, but I can give you some idea of how I approached the problem.

Let yi=xi6y_i = x_i^6, so xi=yi1/6x_i = y_i^{1/6} and y1+y2++yn=n.y_1 + y_2 + \dots + y_n = n. Also, let f(y)=y5/6y1/3f(y) = y^{5/6} - y^{1/3}, and let S=i=1nf(yi).S = \sum_{i = 1}^n f(y_i). Then the problem is to find all nn for which SS is always nonnegative.

It took a lot of work and persistence to even find an nn for which SS could be negative. The results I was getting suggested looking at an example where one of the yiy_i was relatively "large," and the remaining yiy_i were "small." Once I found an example that worked, the rest was just refining the example to get the answer.

Beyond that, there's not much more I can say, except that the answer is much bigger than 2. I can only recommend being tenacious, and keeping things simple. That's what I did.

Jon Haussmann - 7 years, 5 months ago
×

Problem Loading...

Note Loading...

Set Loading...